Which of the following statements about determinants are correct?












-1












$begingroup$



Which of the following statements about determinants are correct?




  1. $det(A^2)>0$, for all invertible matrices $A$


  2. $det(A+A^{-1})=det(A)+dfrac{1}{det(A)}$, for all invertible matrices $A$


  3. $det(vv^T)>0$, for all column vectors $v ≠ 0$


  4. $det(AB^T)=det((A^T)B)$, for all square matrices $A$ and $B$





Which of the statements are correct? I do not feel secure about which of them that is true.



My answer:
My calculations have given me that $2$ and $4$ are true. Am I correct?










share|cite|improve this question











$endgroup$








  • 1




    $begingroup$
    kindly edit your post to include your attempts using mathjax.
    $endgroup$
    – Siong Thye Goh
    Dec 19 '18 at 6:54












  • $begingroup$
    Okey. Sorry I forgot
    $endgroup$
    – Jacob Andreasson
    Dec 19 '18 at 6:55










  • $begingroup$
    What do you know about determinants?
    $endgroup$
    – bubba
    Dec 19 '18 at 6:58






  • 1




    $begingroup$
    Do you know, for example, that $det(AB) = det(A)cdotdet(B)$ ?
    $endgroup$
    – bubba
    Dec 19 '18 at 6:59












  • $begingroup$
    Sorry I have calculate totally wrong
    $endgroup$
    – Jacob Andreasson
    Dec 19 '18 at 7:00
















-1












$begingroup$



Which of the following statements about determinants are correct?




  1. $det(A^2)>0$, for all invertible matrices $A$


  2. $det(A+A^{-1})=det(A)+dfrac{1}{det(A)}$, for all invertible matrices $A$


  3. $det(vv^T)>0$, for all column vectors $v ≠ 0$


  4. $det(AB^T)=det((A^T)B)$, for all square matrices $A$ and $B$





Which of the statements are correct? I do not feel secure about which of them that is true.



My answer:
My calculations have given me that $2$ and $4$ are true. Am I correct?










share|cite|improve this question











$endgroup$








  • 1




    $begingroup$
    kindly edit your post to include your attempts using mathjax.
    $endgroup$
    – Siong Thye Goh
    Dec 19 '18 at 6:54












  • $begingroup$
    Okey. Sorry I forgot
    $endgroup$
    – Jacob Andreasson
    Dec 19 '18 at 6:55










  • $begingroup$
    What do you know about determinants?
    $endgroup$
    – bubba
    Dec 19 '18 at 6:58






  • 1




    $begingroup$
    Do you know, for example, that $det(AB) = det(A)cdotdet(B)$ ?
    $endgroup$
    – bubba
    Dec 19 '18 at 6:59












  • $begingroup$
    Sorry I have calculate totally wrong
    $endgroup$
    – Jacob Andreasson
    Dec 19 '18 at 7:00














-1












-1








-1





$begingroup$



Which of the following statements about determinants are correct?




  1. $det(A^2)>0$, for all invertible matrices $A$


  2. $det(A+A^{-1})=det(A)+dfrac{1}{det(A)}$, for all invertible matrices $A$


  3. $det(vv^T)>0$, for all column vectors $v ≠ 0$


  4. $det(AB^T)=det((A^T)B)$, for all square matrices $A$ and $B$





Which of the statements are correct? I do not feel secure about which of them that is true.



My answer:
My calculations have given me that $2$ and $4$ are true. Am I correct?










share|cite|improve this question











$endgroup$





Which of the following statements about determinants are correct?




  1. $det(A^2)>0$, for all invertible matrices $A$


  2. $det(A+A^{-1})=det(A)+dfrac{1}{det(A)}$, for all invertible matrices $A$


  3. $det(vv^T)>0$, for all column vectors $v ≠ 0$


  4. $det(AB^T)=det((A^T)B)$, for all square matrices $A$ and $B$





Which of the statements are correct? I do not feel secure about which of them that is true.



My answer:
My calculations have given me that $2$ and $4$ are true. Am I correct?







matrices symmetric-matrices






share|cite|improve this question















share|cite|improve this question













share|cite|improve this question




share|cite|improve this question








edited Dec 19 '18 at 7:12









Henno Brandsma

114k348124




114k348124










asked Dec 19 '18 at 6:53









Jacob AndreassonJacob Andreasson

35




35








  • 1




    $begingroup$
    kindly edit your post to include your attempts using mathjax.
    $endgroup$
    – Siong Thye Goh
    Dec 19 '18 at 6:54












  • $begingroup$
    Okey. Sorry I forgot
    $endgroup$
    – Jacob Andreasson
    Dec 19 '18 at 6:55










  • $begingroup$
    What do you know about determinants?
    $endgroup$
    – bubba
    Dec 19 '18 at 6:58






  • 1




    $begingroup$
    Do you know, for example, that $det(AB) = det(A)cdotdet(B)$ ?
    $endgroup$
    – bubba
    Dec 19 '18 at 6:59












  • $begingroup$
    Sorry I have calculate totally wrong
    $endgroup$
    – Jacob Andreasson
    Dec 19 '18 at 7:00














  • 1




    $begingroup$
    kindly edit your post to include your attempts using mathjax.
    $endgroup$
    – Siong Thye Goh
    Dec 19 '18 at 6:54












  • $begingroup$
    Okey. Sorry I forgot
    $endgroup$
    – Jacob Andreasson
    Dec 19 '18 at 6:55










  • $begingroup$
    What do you know about determinants?
    $endgroup$
    – bubba
    Dec 19 '18 at 6:58






  • 1




    $begingroup$
    Do you know, for example, that $det(AB) = det(A)cdotdet(B)$ ?
    $endgroup$
    – bubba
    Dec 19 '18 at 6:59












  • $begingroup$
    Sorry I have calculate totally wrong
    $endgroup$
    – Jacob Andreasson
    Dec 19 '18 at 7:00








1




1




$begingroup$
kindly edit your post to include your attempts using mathjax.
$endgroup$
– Siong Thye Goh
Dec 19 '18 at 6:54






$begingroup$
kindly edit your post to include your attempts using mathjax.
$endgroup$
– Siong Thye Goh
Dec 19 '18 at 6:54














$begingroup$
Okey. Sorry I forgot
$endgroup$
– Jacob Andreasson
Dec 19 '18 at 6:55




$begingroup$
Okey. Sorry I forgot
$endgroup$
– Jacob Andreasson
Dec 19 '18 at 6:55












$begingroup$
What do you know about determinants?
$endgroup$
– bubba
Dec 19 '18 at 6:58




$begingroup$
What do you know about determinants?
$endgroup$
– bubba
Dec 19 '18 at 6:58




1




1




$begingroup$
Do you know, for example, that $det(AB) = det(A)cdotdet(B)$ ?
$endgroup$
– bubba
Dec 19 '18 at 6:59






$begingroup$
Do you know, for example, that $det(AB) = det(A)cdotdet(B)$ ?
$endgroup$
– bubba
Dec 19 '18 at 6:59














$begingroup$
Sorry I have calculate totally wrong
$endgroup$
– Jacob Andreasson
Dec 19 '18 at 7:00




$begingroup$
Sorry I have calculate totally wrong
$endgroup$
– Jacob Andreasson
Dec 19 '18 at 7:00










2 Answers
2






active

oldest

votes


















3












$begingroup$


  • $det (A^2)=det A cdot det A=(det A)^2>0$


  • $det (I+I)=det (2I)=2^n .det I=2^n$ whereas $det I+frac{1}{det I}=2$


  • $vv^T$ has rank $1$, so its eigenvalues are zero [$(n-1)$ times] and its trace and hence $det vv^T=0$

  • $det (AB^T)=det A cdot det B^T=det A cdot det B=det A^T.det B=det(A^TB)$






share|cite|improve this answer











$endgroup$













  • $begingroup$
    so the only one that is true is 4?
    $endgroup$
    – Jacob Andreasson
    Dec 19 '18 at 7:27










  • $begingroup$
    it is A*B$^T$ not (AB)$^T$
    $endgroup$
    – Jacob Andreasson
    Dec 19 '18 at 7:30










  • $begingroup$
    see my answer again. use $det A=det A^T$
    $endgroup$
    – Chinnapparaj R
    Dec 19 '18 at 7:33










  • $begingroup$
    1 and 4 is true then?
    $endgroup$
    – Jacob Andreasson
    Dec 19 '18 at 7:36










  • $begingroup$
    yes...................
    $endgroup$
    – Chinnapparaj R
    Dec 19 '18 at 7:36



















1












$begingroup$

For 1, use that $det(AB)= det(A)det(B)$ to find that $det(A^2) = det(A)^2 > 0$.



For 2, take $A = I $.



For 3, note that the rows of every matrix $vv^T$ are scalar multiples of $v$. Thus, the determinant will be zero for all vectors with more thant $1$ entry.



For 4, use that $det(A^T) = det(A)$, so $det(AB^T) = det((AB^T)^T) = det(BA^T) = det(A^TB)$.






share|cite|improve this answer









$endgroup$














    Your Answer





    StackExchange.ifUsing("editor", function () {
    return StackExchange.using("mathjaxEditing", function () {
    StackExchange.MarkdownEditor.creationCallbacks.add(function (editor, postfix) {
    StackExchange.mathjaxEditing.prepareWmdForMathJax(editor, postfix, [["$", "$"], ["\\(","\\)"]]);
    });
    });
    }, "mathjax-editing");

    StackExchange.ready(function() {
    var channelOptions = {
    tags: "".split(" "),
    id: "69"
    };
    initTagRenderer("".split(" "), "".split(" "), channelOptions);

    StackExchange.using("externalEditor", function() {
    // Have to fire editor after snippets, if snippets enabled
    if (StackExchange.settings.snippets.snippetsEnabled) {
    StackExchange.using("snippets", function() {
    createEditor();
    });
    }
    else {
    createEditor();
    }
    });

    function createEditor() {
    StackExchange.prepareEditor({
    heartbeatType: 'answer',
    autoActivateHeartbeat: false,
    convertImagesToLinks: true,
    noModals: true,
    showLowRepImageUploadWarning: true,
    reputationToPostImages: 10,
    bindNavPrevention: true,
    postfix: "",
    imageUploader: {
    brandingHtml: "Powered by u003ca class="icon-imgur-white" href="https://imgur.com/"u003eu003c/au003e",
    contentPolicyHtml: "User contributions licensed under u003ca href="https://creativecommons.org/licenses/by-sa/3.0/"u003ecc by-sa 3.0 with attribution requiredu003c/au003e u003ca href="https://stackoverflow.com/legal/content-policy"u003e(content policy)u003c/au003e",
    allowUrls: true
    },
    noCode: true, onDemand: true,
    discardSelector: ".discard-answer"
    ,immediatelyShowMarkdownHelp:true
    });


    }
    });














    draft saved

    draft discarded


















    StackExchange.ready(
    function () {
    StackExchange.openid.initPostLogin('.new-post-login', 'https%3a%2f%2fmath.stackexchange.com%2fquestions%2f3046109%2fwhich-of-the-following-statements-about-determinants-are-correct%23new-answer', 'question_page');
    }
    );

    Post as a guest















    Required, but never shown

























    2 Answers
    2






    active

    oldest

    votes








    2 Answers
    2






    active

    oldest

    votes









    active

    oldest

    votes






    active

    oldest

    votes









    3












    $begingroup$


    • $det (A^2)=det A cdot det A=(det A)^2>0$


    • $det (I+I)=det (2I)=2^n .det I=2^n$ whereas $det I+frac{1}{det I}=2$


    • $vv^T$ has rank $1$, so its eigenvalues are zero [$(n-1)$ times] and its trace and hence $det vv^T=0$

    • $det (AB^T)=det A cdot det B^T=det A cdot det B=det A^T.det B=det(A^TB)$






    share|cite|improve this answer











    $endgroup$













    • $begingroup$
      so the only one that is true is 4?
      $endgroup$
      – Jacob Andreasson
      Dec 19 '18 at 7:27










    • $begingroup$
      it is A*B$^T$ not (AB)$^T$
      $endgroup$
      – Jacob Andreasson
      Dec 19 '18 at 7:30










    • $begingroup$
      see my answer again. use $det A=det A^T$
      $endgroup$
      – Chinnapparaj R
      Dec 19 '18 at 7:33










    • $begingroup$
      1 and 4 is true then?
      $endgroup$
      – Jacob Andreasson
      Dec 19 '18 at 7:36










    • $begingroup$
      yes...................
      $endgroup$
      – Chinnapparaj R
      Dec 19 '18 at 7:36
















    3












    $begingroup$


    • $det (A^2)=det A cdot det A=(det A)^2>0$


    • $det (I+I)=det (2I)=2^n .det I=2^n$ whereas $det I+frac{1}{det I}=2$


    • $vv^T$ has rank $1$, so its eigenvalues are zero [$(n-1)$ times] and its trace and hence $det vv^T=0$

    • $det (AB^T)=det A cdot det B^T=det A cdot det B=det A^T.det B=det(A^TB)$






    share|cite|improve this answer











    $endgroup$













    • $begingroup$
      so the only one that is true is 4?
      $endgroup$
      – Jacob Andreasson
      Dec 19 '18 at 7:27










    • $begingroup$
      it is A*B$^T$ not (AB)$^T$
      $endgroup$
      – Jacob Andreasson
      Dec 19 '18 at 7:30










    • $begingroup$
      see my answer again. use $det A=det A^T$
      $endgroup$
      – Chinnapparaj R
      Dec 19 '18 at 7:33










    • $begingroup$
      1 and 4 is true then?
      $endgroup$
      – Jacob Andreasson
      Dec 19 '18 at 7:36










    • $begingroup$
      yes...................
      $endgroup$
      – Chinnapparaj R
      Dec 19 '18 at 7:36














    3












    3








    3





    $begingroup$


    • $det (A^2)=det A cdot det A=(det A)^2>0$


    • $det (I+I)=det (2I)=2^n .det I=2^n$ whereas $det I+frac{1}{det I}=2$


    • $vv^T$ has rank $1$, so its eigenvalues are zero [$(n-1)$ times] and its trace and hence $det vv^T=0$

    • $det (AB^T)=det A cdot det B^T=det A cdot det B=det A^T.det B=det(A^TB)$






    share|cite|improve this answer











    $endgroup$




    • $det (A^2)=det A cdot det A=(det A)^2>0$


    • $det (I+I)=det (2I)=2^n .det I=2^n$ whereas $det I+frac{1}{det I}=2$


    • $vv^T$ has rank $1$, so its eigenvalues are zero [$(n-1)$ times] and its trace and hence $det vv^T=0$

    • $det (AB^T)=det A cdot det B^T=det A cdot det B=det A^T.det B=det(A^TB)$







    share|cite|improve this answer














    share|cite|improve this answer



    share|cite|improve this answer








    edited Dec 19 '18 at 7:32

























    answered Dec 19 '18 at 7:25









    Chinnapparaj RChinnapparaj R

    5,8682928




    5,8682928












    • $begingroup$
      so the only one that is true is 4?
      $endgroup$
      – Jacob Andreasson
      Dec 19 '18 at 7:27










    • $begingroup$
      it is A*B$^T$ not (AB)$^T$
      $endgroup$
      – Jacob Andreasson
      Dec 19 '18 at 7:30










    • $begingroup$
      see my answer again. use $det A=det A^T$
      $endgroup$
      – Chinnapparaj R
      Dec 19 '18 at 7:33










    • $begingroup$
      1 and 4 is true then?
      $endgroup$
      – Jacob Andreasson
      Dec 19 '18 at 7:36










    • $begingroup$
      yes...................
      $endgroup$
      – Chinnapparaj R
      Dec 19 '18 at 7:36


















    • $begingroup$
      so the only one that is true is 4?
      $endgroup$
      – Jacob Andreasson
      Dec 19 '18 at 7:27










    • $begingroup$
      it is A*B$^T$ not (AB)$^T$
      $endgroup$
      – Jacob Andreasson
      Dec 19 '18 at 7:30










    • $begingroup$
      see my answer again. use $det A=det A^T$
      $endgroup$
      – Chinnapparaj R
      Dec 19 '18 at 7:33










    • $begingroup$
      1 and 4 is true then?
      $endgroup$
      – Jacob Andreasson
      Dec 19 '18 at 7:36










    • $begingroup$
      yes...................
      $endgroup$
      – Chinnapparaj R
      Dec 19 '18 at 7:36
















    $begingroup$
    so the only one that is true is 4?
    $endgroup$
    – Jacob Andreasson
    Dec 19 '18 at 7:27




    $begingroup$
    so the only one that is true is 4?
    $endgroup$
    – Jacob Andreasson
    Dec 19 '18 at 7:27












    $begingroup$
    it is A*B$^T$ not (AB)$^T$
    $endgroup$
    – Jacob Andreasson
    Dec 19 '18 at 7:30




    $begingroup$
    it is A*B$^T$ not (AB)$^T$
    $endgroup$
    – Jacob Andreasson
    Dec 19 '18 at 7:30












    $begingroup$
    see my answer again. use $det A=det A^T$
    $endgroup$
    – Chinnapparaj R
    Dec 19 '18 at 7:33




    $begingroup$
    see my answer again. use $det A=det A^T$
    $endgroup$
    – Chinnapparaj R
    Dec 19 '18 at 7:33












    $begingroup$
    1 and 4 is true then?
    $endgroup$
    – Jacob Andreasson
    Dec 19 '18 at 7:36




    $begingroup$
    1 and 4 is true then?
    $endgroup$
    – Jacob Andreasson
    Dec 19 '18 at 7:36












    $begingroup$
    yes...................
    $endgroup$
    – Chinnapparaj R
    Dec 19 '18 at 7:36




    $begingroup$
    yes...................
    $endgroup$
    – Chinnapparaj R
    Dec 19 '18 at 7:36











    1












    $begingroup$

    For 1, use that $det(AB)= det(A)det(B)$ to find that $det(A^2) = det(A)^2 > 0$.



    For 2, take $A = I $.



    For 3, note that the rows of every matrix $vv^T$ are scalar multiples of $v$. Thus, the determinant will be zero for all vectors with more thant $1$ entry.



    For 4, use that $det(A^T) = det(A)$, so $det(AB^T) = det((AB^T)^T) = det(BA^T) = det(A^TB)$.






    share|cite|improve this answer









    $endgroup$


















      1












      $begingroup$

      For 1, use that $det(AB)= det(A)det(B)$ to find that $det(A^2) = det(A)^2 > 0$.



      For 2, take $A = I $.



      For 3, note that the rows of every matrix $vv^T$ are scalar multiples of $v$. Thus, the determinant will be zero for all vectors with more thant $1$ entry.



      For 4, use that $det(A^T) = det(A)$, so $det(AB^T) = det((AB^T)^T) = det(BA^T) = det(A^TB)$.






      share|cite|improve this answer









      $endgroup$
















        1












        1








        1





        $begingroup$

        For 1, use that $det(AB)= det(A)det(B)$ to find that $det(A^2) = det(A)^2 > 0$.



        For 2, take $A = I $.



        For 3, note that the rows of every matrix $vv^T$ are scalar multiples of $v$. Thus, the determinant will be zero for all vectors with more thant $1$ entry.



        For 4, use that $det(A^T) = det(A)$, so $det(AB^T) = det((AB^T)^T) = det(BA^T) = det(A^TB)$.






        share|cite|improve this answer









        $endgroup$



        For 1, use that $det(AB)= det(A)det(B)$ to find that $det(A^2) = det(A)^2 > 0$.



        For 2, take $A = I $.



        For 3, note that the rows of every matrix $vv^T$ are scalar multiples of $v$. Thus, the determinant will be zero for all vectors with more thant $1$ entry.



        For 4, use that $det(A^T) = det(A)$, so $det(AB^T) = det((AB^T)^T) = det(BA^T) = det(A^TB)$.







        share|cite|improve this answer












        share|cite|improve this answer



        share|cite|improve this answer










        answered Dec 19 '18 at 7:31









        Anthony TerAnthony Ter

        37116




        37116






























            draft saved

            draft discarded




















































            Thanks for contributing an answer to Mathematics Stack Exchange!


            • Please be sure to answer the question. Provide details and share your research!

            But avoid



            • Asking for help, clarification, or responding to other answers.

            • Making statements based on opinion; back them up with references or personal experience.


            Use MathJax to format equations. MathJax reference.


            To learn more, see our tips on writing great answers.




            draft saved


            draft discarded














            StackExchange.ready(
            function () {
            StackExchange.openid.initPostLogin('.new-post-login', 'https%3a%2f%2fmath.stackexchange.com%2fquestions%2f3046109%2fwhich-of-the-following-statements-about-determinants-are-correct%23new-answer', 'question_page');
            }
            );

            Post as a guest















            Required, but never shown





















































            Required, but never shown














            Required, but never shown












            Required, but never shown







            Required, but never shown

































            Required, but never shown














            Required, but never shown












            Required, but never shown







            Required, but never shown







            Popular posts from this blog

            Plaza Victoria

            In PowerPoint, is there a keyboard shortcut for bulleted / numbered list?

            How to put 3 figures in Latex with 2 figures side by side and 1 below these side by side images but in...